LSAT and Law School Admissions Forum

Get expert LSAT preparation and law school admissions advice from PowerScore Test Preparation.

 brettb
  • Posts: 14
  • Joined: Mar 29, 2016
|
#22622
Hello,

I was able to quickly eliminate A,C & E.

I correctly answered "B" however, I had trouble fully understanding exactly what answer choice D was saying. Could someone please break this answer choice down in simplified language? I even sort of understand it was talking about necessary & sufficient conditions, but I couldn't quite comprehend or wrap my head around the broad language as it relates to the stimulus.
 Emily Haney-Caron
PowerScore Staff
  • PowerScore Staff
  • Posts: 577
  • Joined: Jan 12, 2012
|
#22627
Hi Brett,

Thanks for the question! D is saying that the argument assumes something is necessary for an interest to be motivating is actually sufficient to show that the interest is motivating. The reason this sounds so confusing is because it has nothing to do with this stimulus, and so it doesn't make sense in this context. The wording can trip you up, for sure, but the key here is to identify that it is referencing the common flaw of mistaking a sufficient condition for a necessary one - once you've gotten to that point, you can identify that it is not a flaw we see in this question, without needing to understand the nitty gritty of what D is saying.
 jessicamorehead
  • Posts: 84
  • Joined: Jul 07, 2017
|
#45317
I correctly chose answer choice B. I noticed that the supporting premise used "can," while the conclusion was definite with an "are." I eliminated D because it was describing a conditional flaw, but I decided that there cannot be a conditional flaw here since you cannot diagram conditional logic with words like "can." Conditionality is 100%, but "can" leaves open the possibility of not happening, therefore it is not conditional. Is my logic okay here?
 Alex Bodaken
PowerScore Staff
  • PowerScore Staff
  • Posts: 136
  • Joined: Feb 21, 2018
|
#45747
jessicamorehead,

Thanks for the question! I think ruling out (D) the way you did might be okay, but I would not be too quick to eliminate answers based on one or two words (the LSAT can sometimes try and trick us that way!) Alternatively, you could also try and think about what the answer choices are arguing...choice (D) argues that it "ignores the possibility that what is taken to be necessary for a certain interest to be a motivation actually suffices to show that that interest is a motivation"...but not only does it not ignore this possibility, this is the exact argument the author makes! She is saying exactly that - that a condition that may be necessary (self-interest) for a motivation is sufficient to show it is a motivation. So this can't be the flaw, because the author distinctively does not ignore this possibility.

Hope that helps!
Alex
 akanshalsat
  • Posts: 104
  • Joined: Dec 20, 2017
|
#48630
Hello :)

I got this question correct, but could someone please explain a little more in depth what D is saying and how it relates to the stimulus? I think I have trouble with answer choices that describe conditionality in which it says things like "what is taken to be necessary, is actually sufficient" or "what is sufficient an be taken as necessary" I think it just confuses my brain and that causes me to just skip those answer choices out of helplessness, not because I think they are wrong or right.

Thank you so much!
 Adam Tyson
PowerScore Staff
  • PowerScore Staff
  • Posts: 5153
  • Joined: Apr 14, 2011
|
#49083
Those kinds of answers are describing either Mistaken Reversals or Mistaken Negations, both of which are errors of conditional reasoning. You'll know that you are looking at a conditional argument when it either contains classic conditional reasoning indicators like "if" and "only" and "unless", or when you are able to paraphrase the argument using an "if...then" structure without altering the meaning of the argument.

If the argument is conditional, and if you are told that it has a flaw, then the answer will probably use the sort of language you are talking about to describe that flaw. If the argument is not conditional, then answers like that are there just to distract and confuse you. Don't fall for them!

For example, consider this argument:

"if someone is altruistic, that person is not motivated by self-interest. Jim is not motivated by self-interest, so he is altruistic"

This argument contains a mistaken reversal - the author has treated a necessary condition (not being motivated by self-interest, which is necessary when one is altruistic) as if it was sufficient (to prove altruism).

Conditional reasoning is a powerful and prominent feature of the LSAT, in both LR and LG. If you are having trouble with it, you will definitely want to study it! There's a lot of discussion of it here in this forum, and in many articles on our blog, and of course in the LR Bible and many of our other publications. Getting comfortable and familiar with it can make a huge, huge impact on your score, so look into it! You won't regret it!
 LSAT2018
  • Posts: 242
  • Joined: Jan 10, 2018
|
#49094
Alex Bodaken wrote:jessicamorehead,

Thanks for the question! I think ruling out (D) the way you did might be okay, but I would not be too quick to eliminate answers based on one or two words (the LSAT can sometimes try and trick us that way!) Alternatively, you could also try and think about what the answer choices are arguing...choice (D) argues that it "ignores the possibility that what is taken to be necessary for a certain interest to be a motivation actually suffices to show that that interest is a motivation"...but not only does it not ignore this possibility, this is the exact argument the author makes! She is saying exactly that - that a condition that may be necessary (self-interest) for a motivation is sufficient to show it is a motivation. So this can't be the flaw, because the author distinctively does not ignore this possibility.

Hope that helps!
Alex

Can I get clarification on this explanation as it doesn't seem to say the same thing as the explanation that came before it? So the necessary condition is self interest and sufficient condition is motivation? I`m not understanding how the author is doing this.
 Adam Tyson
PowerScore Staff
  • PowerScore Staff
  • Posts: 5153
  • Joined: Apr 14, 2011
|
#49137
There is a premise that could be diagrammed conditionally, LSAT2018. It is:

Altruistic :arrow: CAN be described as self-interest

(I would not diagram this, because the CAN means it isn't truly necessary, but one could take this approach)

But the conclusion is not a conditional one, which might be "if something could be described as self-interest, then it is altruistic", a mistaken reversal. Rather, the flaw is in the author's treatment of something that MAY be true as something that MUST be true. Alex's response talks about a condition that MAY be necessary. So, we are saying the same thing, but in different ways (because when one student doesn't seem to fully grasp it the way one of us explained it, we want to try it from another angle to see if that works better for them). There's no conflict here - this is not a conditional argument, although it contains a conditional premise.

Sorry if that confused you!
User avatar
 jrschultz14
  • Posts: 9
  • Joined: Aug 02, 2021
|
#90032
Hi, I am having trouble wrapping my mind around why AC B was correct -- could you help me understand where my thought process is breaking down? After reading the stimulus, I separated the Conclusion as "all actions are motivated by self-interest" and the support as "any action that is apparently altruistic can be described in terms of self-interest."

When looking at the C and P I anticipated that the AC would have something to do with the fact that "described in terms of self-interest" =/= "motivated by self-interest." Just because you can describe something as self-interest doesn't mean it is self-interest.

AC B then says that the argument "takes evidence showing merely that the argument could be true to constitute evidence that the conclusion is in fact true," but when reading the P and C, I only see evidence that the P could be true, not evidence that the argument as a whole could be true. By "argument" do they just mean "premise?" Because after reading the stimulus, I didn't feel like they had provided any evidence that their argument could be true.

This may be confusing. If this doesn't make sense, please let me know. Thanks!
User avatar
 Beatrice Brown
PowerScore Staff
  • PowerScore Staff
  • Posts: 75
  • Joined: Jun 30, 2021
|
#90066
Hi JR! Thanks for your question, and happy to help you out with this :)

Great job identifying the premise and the conclusion of this argument. Let me put this another way and re-phrase the argument, because this will be helpful to see why answer choice (B) is correct: the author argues that all actions are motivated by self-interest because we can describe altruistic acts in terms of self-interest. The author then gives an example of how we can describe helping someone in terms of self-interest.

The reason I italicized "are" and "can" in my summary of the argument is that this is the crux of the flaw: the author makes a conclusion about what actions must be based on how we can describe them. But just because we can describe a specific type of action (altruistic acts) in terms of self-interest does not mean that all actions must be motivated by self-interest. Perhaps we can describe actions in non-self-interested ways; if there are multiple ways to describe an action, then we cannot conclude anything about what those actions must be. Note that your prephrase for this question definitely touches on this disconnect between "are" and "can" :)

Turning to answer choice (B), this matches our prephrase. Answer choice (B) tells us that the argument uses evidence that shows the conclusion could be true to argue that the conclusion must be true. This is what the stimulus does: the author uses evidence that shows that we can describe certain actions in terms of self-interest (using an example of a particular altruistic act) to prove the conclusion that all actions must be self-interested. But providing one example just shows us that the conclusion could be true, not that it must be true; the example of being able to describe helping someone in terms of self-interest means actions may be motivated by self-interest, but not that they must be.

To clarify your specifc question about answer choice (B) a bit further: the author did provide evidence that their conclusion could be true, which is the example of being able to describe helping someone in terms of self-interest. The issue is this evidence does not mean that the conclusion must be true. When the answer choice uses the word "argument," they do not mean "premise." Instead, the argument refers to the stimulus as a whole. Since the stimulus itself does take evidence of what can be true as proof of what must be true, this is the correct answer choice.

I hope this helps, and please let me know if you have any other questions!

Get the most out of your LSAT Prep Plus subscription.

Analyze and track your performance with our Testing and Analytics Package.